How to write a polynomial in mod p

The name of the pictureThe name of the pictureThe name of the pictureClash Royale CLAN TAG#URR8PPP











up vote
3
down vote

favorite












Consider the polynomial $ x^3 - 3 x^2 +2 x -1$
How can this polynomial be written in mod 3 ?



What confuses me is that I thought we don't change any power . If we have to also consider the powers then will the leading coefficient be equal to zero then.







share|cite|improve this question


























    up vote
    3
    down vote

    favorite












    Consider the polynomial $ x^3 - 3 x^2 +2 x -1$
    How can this polynomial be written in mod 3 ?



    What confuses me is that I thought we don't change any power . If we have to also consider the powers then will the leading coefficient be equal to zero then.







    share|cite|improve this question
























      up vote
      3
      down vote

      favorite









      up vote
      3
      down vote

      favorite











      Consider the polynomial $ x^3 - 3 x^2 +2 x -1$
      How can this polynomial be written in mod 3 ?



      What confuses me is that I thought we don't change any power . If we have to also consider the powers then will the leading coefficient be equal to zero then.







      share|cite|improve this question














      Consider the polynomial $ x^3 - 3 x^2 +2 x -1$
      How can this polynomial be written in mod 3 ?



      What confuses me is that I thought we don't change any power . If we have to also consider the powers then will the leading coefficient be equal to zero then.









      share|cite|improve this question













      share|cite|improve this question




      share|cite|improve this question








      edited Aug 9 at 23:13

























      asked Aug 9 at 22:37









      Rosa1

      686




      686




















          2 Answers
          2






          active

          oldest

          votes

















          up vote
          2
          down vote



          accepted










          If $x$ is an integer, then
          beginalign*
          &x^3-3x^2+2x-1\[4pt]
          equiv;&x^3-0x^2+2x-1;(textmod;3)\[4pt]
          equiv;&x^3+2x-1;(textmod;3)\[4pt]
          equiv;&x+2x-1;(textmod;3)
          qquadtext[since $x^3equiv x;(textmod;3)$]\[4pt]
          equiv;&3x-1;(textmod;3)\[4pt]
          equiv;&0x-1;(textmod;3)\[4pt]
          equiv;&-1;(textmod;3)\[4pt]
          equiv;&2;(textmod;3)\[4pt]
          endalign*
          If $x$ is just an indeterminate, then you can't replace $x^3$ by $x$, but you can replace coefficients by their residues, mod $3$, hence, with that interpretation, you can get
          beginalign*
          &x^3-3x^2+2x-1
          qquadqquadqquadqquadqquad;;;;;;,
          \[4pt]
          equiv;&x^3-0x^2+2x-1;(textmod;3)\[4pt]
          equiv;&x^3+2x-1;(textmod;3)\[4pt]
          equiv;&x^3+2x+2;(textmod;3)\[4pt]
          endalign*






          share|cite|improve this answer






















          • actually in my note it is right up to your 4th line before "since" but I could not understand how we get x from $ x^3 $ although we should not change the powers
            – Rosa1
            Aug 9 at 23:11










          • If $x$ is intended as an indeterminate, then you can't change powers. If $x$ is just an unknown integer, then $x^3equiv x;(textmod;3)$ by Fermat's little Theorem, so with that interpretation, $x^3$ can be replaced by $x$. Thus, it depends on the intended interpretation of $x$. If you really meant polynomial, then $x$ should be regarded as an indeterminate, so in that case, you can't replace $x^3$ by $x$. In my answer, I showed both interpretations.
            – quasi
            Aug 9 at 23:21


















          up vote
          4
          down vote














          What confuses me is that I thought we don't change any power . If we have to also consider the powers then will the leading coefficient be equal to zero then.




          Your first thought is correct: we don't change any power. Even though $3$ is in the exponent, we don't take the exponents mod 3, but only the coefficients.



          So, taking a polynomial mod 3 just means taking each coefficient mod 3 (and leaving the exponents). For example, $6x^3 + 4x^2 - 2 mod 3$ would be $0x^3 + x^2 + 1$, which is the same as $x^2 + 1$ since we can drop $0$s. This is because $6 mod 3$ is $0$, $4 mod 3$ is $1$, and $-2 mod 3$ is $1$.






          share|cite|improve this answer




















            Your Answer




            StackExchange.ifUsing("editor", function ()
            return StackExchange.using("mathjaxEditing", function ()
            StackExchange.MarkdownEditor.creationCallbacks.add(function (editor, postfix)
            StackExchange.mathjaxEditing.prepareWmdForMathJax(editor, postfix, [["$", "$"], ["\\(","\\)"]]);
            );
            );
            , "mathjax-editing");

            StackExchange.ready(function()
            var channelOptions =
            tags: "".split(" "),
            id: "69"
            ;
            initTagRenderer("".split(" "), "".split(" "), channelOptions);

            StackExchange.using("externalEditor", function()
            // Have to fire editor after snippets, if snippets enabled
            if (StackExchange.settings.snippets.snippetsEnabled)
            StackExchange.using("snippets", function()
            createEditor();
            );

            else
            createEditor();

            );

            function createEditor()
            StackExchange.prepareEditor(
            heartbeatType: 'answer',
            convertImagesToLinks: true,
            noModals: false,
            showLowRepImageUploadWarning: true,
            reputationToPostImages: 10,
            bindNavPrevention: true,
            postfix: "",
            noCode: true, onDemand: true,
            discardSelector: ".discard-answer"
            ,immediatelyShowMarkdownHelp:true
            );



            );













             

            draft saved


            draft discarded


















            StackExchange.ready(
            function ()
            StackExchange.openid.initPostLogin('.new-post-login', 'https%3a%2f%2fmath.stackexchange.com%2fquestions%2f2877775%2fhow-to-write-a-polynomial-in-mod-p%23new-answer', 'question_page');

            );

            Post as a guest






























            2 Answers
            2






            active

            oldest

            votes








            2 Answers
            2






            active

            oldest

            votes









            active

            oldest

            votes






            active

            oldest

            votes








            up vote
            2
            down vote



            accepted










            If $x$ is an integer, then
            beginalign*
            &x^3-3x^2+2x-1\[4pt]
            equiv;&x^3-0x^2+2x-1;(textmod;3)\[4pt]
            equiv;&x^3+2x-1;(textmod;3)\[4pt]
            equiv;&x+2x-1;(textmod;3)
            qquadtext[since $x^3equiv x;(textmod;3)$]\[4pt]
            equiv;&3x-1;(textmod;3)\[4pt]
            equiv;&0x-1;(textmod;3)\[4pt]
            equiv;&-1;(textmod;3)\[4pt]
            equiv;&2;(textmod;3)\[4pt]
            endalign*
            If $x$ is just an indeterminate, then you can't replace $x^3$ by $x$, but you can replace coefficients by their residues, mod $3$, hence, with that interpretation, you can get
            beginalign*
            &x^3-3x^2+2x-1
            qquadqquadqquadqquadqquad;;;;;;,
            \[4pt]
            equiv;&x^3-0x^2+2x-1;(textmod;3)\[4pt]
            equiv;&x^3+2x-1;(textmod;3)\[4pt]
            equiv;&x^3+2x+2;(textmod;3)\[4pt]
            endalign*






            share|cite|improve this answer






















            • actually in my note it is right up to your 4th line before "since" but I could not understand how we get x from $ x^3 $ although we should not change the powers
              – Rosa1
              Aug 9 at 23:11










            • If $x$ is intended as an indeterminate, then you can't change powers. If $x$ is just an unknown integer, then $x^3equiv x;(textmod;3)$ by Fermat's little Theorem, so with that interpretation, $x^3$ can be replaced by $x$. Thus, it depends on the intended interpretation of $x$. If you really meant polynomial, then $x$ should be regarded as an indeterminate, so in that case, you can't replace $x^3$ by $x$. In my answer, I showed both interpretations.
              – quasi
              Aug 9 at 23:21















            up vote
            2
            down vote



            accepted










            If $x$ is an integer, then
            beginalign*
            &x^3-3x^2+2x-1\[4pt]
            equiv;&x^3-0x^2+2x-1;(textmod;3)\[4pt]
            equiv;&x^3+2x-1;(textmod;3)\[4pt]
            equiv;&x+2x-1;(textmod;3)
            qquadtext[since $x^3equiv x;(textmod;3)$]\[4pt]
            equiv;&3x-1;(textmod;3)\[4pt]
            equiv;&0x-1;(textmod;3)\[4pt]
            equiv;&-1;(textmod;3)\[4pt]
            equiv;&2;(textmod;3)\[4pt]
            endalign*
            If $x$ is just an indeterminate, then you can't replace $x^3$ by $x$, but you can replace coefficients by their residues, mod $3$, hence, with that interpretation, you can get
            beginalign*
            &x^3-3x^2+2x-1
            qquadqquadqquadqquadqquad;;;;;;,
            \[4pt]
            equiv;&x^3-0x^2+2x-1;(textmod;3)\[4pt]
            equiv;&x^3+2x-1;(textmod;3)\[4pt]
            equiv;&x^3+2x+2;(textmod;3)\[4pt]
            endalign*






            share|cite|improve this answer






















            • actually in my note it is right up to your 4th line before "since" but I could not understand how we get x from $ x^3 $ although we should not change the powers
              – Rosa1
              Aug 9 at 23:11










            • If $x$ is intended as an indeterminate, then you can't change powers. If $x$ is just an unknown integer, then $x^3equiv x;(textmod;3)$ by Fermat's little Theorem, so with that interpretation, $x^3$ can be replaced by $x$. Thus, it depends on the intended interpretation of $x$. If you really meant polynomial, then $x$ should be regarded as an indeterminate, so in that case, you can't replace $x^3$ by $x$. In my answer, I showed both interpretations.
              – quasi
              Aug 9 at 23:21













            up vote
            2
            down vote



            accepted







            up vote
            2
            down vote



            accepted






            If $x$ is an integer, then
            beginalign*
            &x^3-3x^2+2x-1\[4pt]
            equiv;&x^3-0x^2+2x-1;(textmod;3)\[4pt]
            equiv;&x^3+2x-1;(textmod;3)\[4pt]
            equiv;&x+2x-1;(textmod;3)
            qquadtext[since $x^3equiv x;(textmod;3)$]\[4pt]
            equiv;&3x-1;(textmod;3)\[4pt]
            equiv;&0x-1;(textmod;3)\[4pt]
            equiv;&-1;(textmod;3)\[4pt]
            equiv;&2;(textmod;3)\[4pt]
            endalign*
            If $x$ is just an indeterminate, then you can't replace $x^3$ by $x$, but you can replace coefficients by their residues, mod $3$, hence, with that interpretation, you can get
            beginalign*
            &x^3-3x^2+2x-1
            qquadqquadqquadqquadqquad;;;;;;,
            \[4pt]
            equiv;&x^3-0x^2+2x-1;(textmod;3)\[4pt]
            equiv;&x^3+2x-1;(textmod;3)\[4pt]
            equiv;&x^3+2x+2;(textmod;3)\[4pt]
            endalign*






            share|cite|improve this answer














            If $x$ is an integer, then
            beginalign*
            &x^3-3x^2+2x-1\[4pt]
            equiv;&x^3-0x^2+2x-1;(textmod;3)\[4pt]
            equiv;&x^3+2x-1;(textmod;3)\[4pt]
            equiv;&x+2x-1;(textmod;3)
            qquadtext[since $x^3equiv x;(textmod;3)$]\[4pt]
            equiv;&3x-1;(textmod;3)\[4pt]
            equiv;&0x-1;(textmod;3)\[4pt]
            equiv;&-1;(textmod;3)\[4pt]
            equiv;&2;(textmod;3)\[4pt]
            endalign*
            If $x$ is just an indeterminate, then you can't replace $x^3$ by $x$, but you can replace coefficients by their residues, mod $3$, hence, with that interpretation, you can get
            beginalign*
            &x^3-3x^2+2x-1
            qquadqquadqquadqquadqquad;;;;;;,
            \[4pt]
            equiv;&x^3-0x^2+2x-1;(textmod;3)\[4pt]
            equiv;&x^3+2x-1;(textmod;3)\[4pt]
            equiv;&x^3+2x+2;(textmod;3)\[4pt]
            endalign*







            share|cite|improve this answer














            share|cite|improve this answer



            share|cite|improve this answer








            edited Aug 9 at 22:59

























            answered Aug 9 at 22:52









            quasi

            33.9k22461




            33.9k22461











            • actually in my note it is right up to your 4th line before "since" but I could not understand how we get x from $ x^3 $ although we should not change the powers
              – Rosa1
              Aug 9 at 23:11










            • If $x$ is intended as an indeterminate, then you can't change powers. If $x$ is just an unknown integer, then $x^3equiv x;(textmod;3)$ by Fermat's little Theorem, so with that interpretation, $x^3$ can be replaced by $x$. Thus, it depends on the intended interpretation of $x$. If you really meant polynomial, then $x$ should be regarded as an indeterminate, so in that case, you can't replace $x^3$ by $x$. In my answer, I showed both interpretations.
              – quasi
              Aug 9 at 23:21

















            • actually in my note it is right up to your 4th line before "since" but I could not understand how we get x from $ x^3 $ although we should not change the powers
              – Rosa1
              Aug 9 at 23:11










            • If $x$ is intended as an indeterminate, then you can't change powers. If $x$ is just an unknown integer, then $x^3equiv x;(textmod;3)$ by Fermat's little Theorem, so with that interpretation, $x^3$ can be replaced by $x$. Thus, it depends on the intended interpretation of $x$. If you really meant polynomial, then $x$ should be regarded as an indeterminate, so in that case, you can't replace $x^3$ by $x$. In my answer, I showed both interpretations.
              – quasi
              Aug 9 at 23:21
















            actually in my note it is right up to your 4th line before "since" but I could not understand how we get x from $ x^3 $ although we should not change the powers
            – Rosa1
            Aug 9 at 23:11




            actually in my note it is right up to your 4th line before "since" but I could not understand how we get x from $ x^3 $ although we should not change the powers
            – Rosa1
            Aug 9 at 23:11












            If $x$ is intended as an indeterminate, then you can't change powers. If $x$ is just an unknown integer, then $x^3equiv x;(textmod;3)$ by Fermat's little Theorem, so with that interpretation, $x^3$ can be replaced by $x$. Thus, it depends on the intended interpretation of $x$. If you really meant polynomial, then $x$ should be regarded as an indeterminate, so in that case, you can't replace $x^3$ by $x$. In my answer, I showed both interpretations.
            – quasi
            Aug 9 at 23:21





            If $x$ is intended as an indeterminate, then you can't change powers. If $x$ is just an unknown integer, then $x^3equiv x;(textmod;3)$ by Fermat's little Theorem, so with that interpretation, $x^3$ can be replaced by $x$. Thus, it depends on the intended interpretation of $x$. If you really meant polynomial, then $x$ should be regarded as an indeterminate, so in that case, you can't replace $x^3$ by $x$. In my answer, I showed both interpretations.
            – quasi
            Aug 9 at 23:21











            up vote
            4
            down vote














            What confuses me is that I thought we don't change any power . If we have to also consider the powers then will the leading coefficient be equal to zero then.




            Your first thought is correct: we don't change any power. Even though $3$ is in the exponent, we don't take the exponents mod 3, but only the coefficients.



            So, taking a polynomial mod 3 just means taking each coefficient mod 3 (and leaving the exponents). For example, $6x^3 + 4x^2 - 2 mod 3$ would be $0x^3 + x^2 + 1$, which is the same as $x^2 + 1$ since we can drop $0$s. This is because $6 mod 3$ is $0$, $4 mod 3$ is $1$, and $-2 mod 3$ is $1$.






            share|cite|improve this answer
























              up vote
              4
              down vote














              What confuses me is that I thought we don't change any power . If we have to also consider the powers then will the leading coefficient be equal to zero then.




              Your first thought is correct: we don't change any power. Even though $3$ is in the exponent, we don't take the exponents mod 3, but only the coefficients.



              So, taking a polynomial mod 3 just means taking each coefficient mod 3 (and leaving the exponents). For example, $6x^3 + 4x^2 - 2 mod 3$ would be $0x^3 + x^2 + 1$, which is the same as $x^2 + 1$ since we can drop $0$s. This is because $6 mod 3$ is $0$, $4 mod 3$ is $1$, and $-2 mod 3$ is $1$.






              share|cite|improve this answer






















                up vote
                4
                down vote










                up vote
                4
                down vote










                What confuses me is that I thought we don't change any power . If we have to also consider the powers then will the leading coefficient be equal to zero then.




                Your first thought is correct: we don't change any power. Even though $3$ is in the exponent, we don't take the exponents mod 3, but only the coefficients.



                So, taking a polynomial mod 3 just means taking each coefficient mod 3 (and leaving the exponents). For example, $6x^3 + 4x^2 - 2 mod 3$ would be $0x^3 + x^2 + 1$, which is the same as $x^2 + 1$ since we can drop $0$s. This is because $6 mod 3$ is $0$, $4 mod 3$ is $1$, and $-2 mod 3$ is $1$.






                share|cite|improve this answer













                What confuses me is that I thought we don't change any power . If we have to also consider the powers then will the leading coefficient be equal to zero then.




                Your first thought is correct: we don't change any power. Even though $3$ is in the exponent, we don't take the exponents mod 3, but only the coefficients.



                So, taking a polynomial mod 3 just means taking each coefficient mod 3 (and leaving the exponents). For example, $6x^3 + 4x^2 - 2 mod 3$ would be $0x^3 + x^2 + 1$, which is the same as $x^2 + 1$ since we can drop $0$s. This is because $6 mod 3$ is $0$, $4 mod 3$ is $1$, and $-2 mod 3$ is $1$.







                share|cite|improve this answer












                share|cite|improve this answer



                share|cite|improve this answer










                answered Aug 9 at 22:46









                6005

                34.9k750124




                34.9k750124



























                     

                    draft saved


                    draft discarded















































                     


                    draft saved


                    draft discarded














                    StackExchange.ready(
                    function ()
                    StackExchange.openid.initPostLogin('.new-post-login', 'https%3a%2f%2fmath.stackexchange.com%2fquestions%2f2877775%2fhow-to-write-a-polynomial-in-mod-p%23new-answer', 'question_page');

                    );

                    Post as a guest













































































                    Comments

                    Popular posts from this blog

                    Long meetings (6-7 hours a day): Being “babysat” by supervisor

                    Is the Concept of Multiple Fantasy Races Scientifically Flawed? [closed]

                    Confectionery